Please answer quick!!! Find the range of the data set represented by this box plot.

80
76
40
56

Please Answer Quick!!! Find The Range Of The Data Set Represented By This Box Plot. 80 76 40 56

Answers

Answer 1

Answer:

highest value (H)= 80

lowest value (L)= 40

range (R)=?

now using formula,

Range (R)=H-L

=80-40

=40

therefore range (R)=40


Related Questions

What is the first step in mathematical induction?

Answers

Answer:

Show that the statement is true for n=1

Step-by-step explanation:

Hey,

Show that the statement is true for n=1

You can check my other answer there which explains a little bit more the ideas.

https://brainly.com/question/17162256

thank you

what are the comparison symbols for 5/6 and 2/5, 4/10 and 7/8, and 3/12 and 1/4

Answer like this: Example

=
<
>

Answers

Answer:

5/6 > 2/44/10 < 7/83/12 = 1/4

Step-by-step explanation:

The comparison will be the same if you subtract the right side and compare to zero:

  a/b ?? c/d . . . . . . . using ?? for the unknown comparison symbol

  a/b - c/d ?? 0 . . . . subtract the fraction on the right

  (ad -bc)/bd ?? 0 . . . combine the two fractions

  ad - bc ?? 0 . . . . . . multiply by bd to make the job easier

__

5/6 and 2/5

  5(5) -6(2) = 25 -12 > 0   ⇒   5/6 > 2/5

4/10 and 7/8

  4(8) -10(7) = 48 - 70 < 0   ⇒   4/10 < 7/8

3/12 and 1/4

  3(4) -12(1) = 0   ⇒   3/12 = 1/4

_____

Of course, you can use your calculator (or your memory) to change each of these to a decimal equivalent. The comparison should be easy at that point.

  0.833 > 0.400

  0.400 < 0.875

  0.250 = 0.250

Use the number line below, where RS = 6y + 2, ST = 3y + 7, and RT = 14y - 11.
a. What is the value of y?
b. Find RS, ST, and RT.

Answers

Answer:

a) y = 4

b) RS = 26, ST = 19, RT = 45

Step-by-step explanation:

From the line given, the following vector equation is true, RS + ST = RT since R, S and T lies in the same straight line.

Given RS = 6y + 2, ST = 3y + 7, and RT = 14y - 11

On substituting this values into the equation above we will have;

6y+2+(3y+7) = 14y-11

6y+2+3y+7 = 14y-11

Collect the like terms

6y+3y-14y = -11-7-2

9y-14y = -20

-5y = -20

y = 20/5

y = 4

Since RS = 6y + 2

RS = 6(4)+2

RS = 24+2

RS = 26

ST = 3y + 7

ST = 3(4)+7

ST = 12+7

ST = 19

Also, RT = 14y - 11

RT = 14(4)-11

RT = 56-11

RT = 45

12-(3-9) 3*3 help please

Answers

Step-by-step explanation:

42 is your answer according to bodmas

find the value of x? please help​

Answers

Answer:

49

Step-by-step explanation:

With these types of problems, you have to subtract the outer and inner values and then divide by 2. So, (125-27)/2 = 49. Hope this helps!

the value of x is 49

Find the probability that when a couple has four ​children, at least one of them is a boy. ​(Assume that boys and girls are equally​ likely.)

Answers

Answer:

The probability that at least, one of the four children the couple has is a boy is 0.8.

Step-by-step explanation:

Given that boys and girls are equally likely, we want to find the probability of having at least, one boy, from four children..

Note that it is possible to have the following for 4 children:

1. 4 boys, 0 girls

2. 3 boys, 1 girl

3. 2 boys, 2 girls

4. 1 boy, 3 girls

5. 0 boys, 4 girls.

To have at least, one boy, out of the 5 options, only 4 is possible.

1. 4 boys, 0 girls.........YES

2. 3 boys, 1 girl ...........YES

3. 2 boys, 2 girls.........YES

4. 1 boy, 3 girls.............YES

5. 0 boys, 4 girls..........NO

The probability is therefore,

(Probability of event = 4) ÷ (Total possible outcome = 5)

P = 4/5 = 0.8

_Thirty-two holes are drilled in rows on a metal block. The number of rows is more than the number of holes in each

row. Find the number of row. (a)7 (b)25(c)67

(d)4 (e) 12
_​

Answers

Answer:

D

Step-by-step explanation:

Let the number of rows be x

And the numbers of holes in each be y

xy = 32

x and y must be factors of 32

From options stated

4 is the only factor of 32

Hence option D is correct

9. Find the mean of the following data :
Х
8
10
12
20
16
F
2
3
7
2
5​

Answers

Answer:

[tex] \boxed{13.15}[/tex]

Step-by-step explanation:

( See the attached picture )

Now,

Mean = [tex] \mathsf{\frac{Σfx}{n} }[/tex]

[tex] \mathsf{ = \frac{250}{19} }[/tex]

[tex] \mathsf{ = 13.15}[/tex]

------------------------------------------------------------------------

In the case of repeated data , follow the steps given below to calculate the mean :

Draw a table with 3 columnsWrite down the items ( x ) in ascending or descending order in the first column and the corresponding frequencies in the second column.Find the product of each item and it's frequency ( fx ) and write in the third column.Find the total of f column and fx column.Divide the sum of fx by the sum of f ( total number of items ) , the quotient is the required mean.

Hope I helped!

Best regards!

A piece of buttered toast falls to the floor 17 times. The toast landed buttered side up 6 times. What is the probability that the toast lands buttered side down?

Answers

Step-by-step explanation:

Given that,

A piece of buttered toast falls to the floor 17 times. The toast landed buttered side up 6 times.

It means that the total number of outcomes are 17

We need to find the probability that the toast lands buttered side down. Favourable oucome is 17-6 = 11

So, probability is given by :

[tex]P(E)=\dfrac{\text{favourable outcomes}}{\text{total no of outcomes}}[/tex]

[tex]P(E)=\dfrac{11}{17}[/tex]

So, the  probability that the toast lands buttered side down is 11/17.

Emma rents a car from a company that rents cars by the hour. She has to pay an initial fee of $75, and then they charge her $9 per hour. Write an equation for the total cost if Emma rents the car for ℎ hours. If Emma has budgeted $250 for the rental cars, how many hours can she rent the car? Assume the car cannot be rented for part of an hour.

Answers

Y = 9h + 75. She can rent the car for 19 hours. The math for determining the amount of hours she can rent the car is 250 minus 75 is 175. 175 divided by 9 is 19.44. Assuming she can’t rent for part of an hour, it would be 19.

(x+1)(x−1)(x−5)=0 HELP

Answers

Answer:

x³ - 5x² - x + 5

Step-by-step explanation:

(x+1)(x-1)(x-5) = 0

fisrt step:

(x+1)(x-1) = x*x + x*-1 + 1*x + 1*-1 = x² - x + x - 1 = x² - 1

then:

(x+1)(x-1)(x-5) = (x²-1)(x-5)

(x²-1)(x-5) = x²*x + x²*-5 -1*x -1*-5 = x³ - 5x² - x + 5

As a bowling instructor, you calculate your students' averages during tournaments. In 5 games, one bowler had the following scores: 143, 156, 172, 133, and 167. What was that bowler's average?

Answers

Answer:

154.2

Step-by-step explanation:

143 plus

156 plus

172 plus

133 plus

167 = 771

divide by 5 equals 154.2

idk how to put the picture but can someone just tell me the points where the two dots go plz will give good rate nd say thx plz answer fast

Graph -8x-y=8

Answers

Answer:

(-1,0) and (0,-8)

Step-by-step explanation:

Hey there!

Well first we’ll graph -8x - y = 8,

Look at the image below

By lookig at the image below we can tell the 2 points are at,

(-1,0) and (0,-8)

Hope this helps :)

If your starting salary is $40000 and you receive a 3% increase at the end of every year, what is the total amount, in dollars, you will earn one the first 16 years that you work

Answers

Answer:

Total amount in dollars= $64614.00

Step-by-step explanation:

Initial starting salary is $40000.

Rate of increase is 3%

Number of years is 16 years

The salary is compounded yearly.

Amount A after 16 years is given as

A= p (1+r/n)^ (nt)

A=40000(1+0.03/16)^(16*16)

A= 40000(1.001875)^(256)

A=40000(1.61534824)

A= 64613.92959

Total amount in dollars= $64614.00

Answer: the answer is $806275

Step-by-step explanation:

A p e x

nick cut a circular cookie into 5 equal slices. what is the angle measure of each slice?

Answers

Using concepts of circles, it is found that the angle measure of each slice is of 72º.

--------------------------------------------

The cookies have circular formats.A complete circle, which is the format of a cookie, has an angular measure of 360º.If it is divided into a number n of equal slices, the angles will be 360 divided by n.

--------------------------------------------

5 equal slices, thus:

[tex]\frac{360}{5} = 72[/tex]

The angle measure of each slice is of 72º.

A similar problem is given at https://brainly.com/question/16746988

A cardboard box without a lid is to be made with a volume of 4 ft 3 . Find the dimensions of the box that requires the least amount of cardboard.

Answers

Answer:

2ft by 2ft by 1 ft

Step-by-step explanation:

Total surface of the cardboard box is expressed as S = 2LW + 2WH + 2LH where L is the length of the box, W is the width and H is the height of the box. Since the cardboard box is without a lid, then the total surface area will be expressed as;

S  = lw+2wh+2lh ... 1

Given the volume V = lwh = 4ft³ ... 2

From equation 2;

h = 4/lw

Substituting into r[equation 1;

S = lw + 2w(4/lw)+ 2l(4/lw)

S = lw+8/l+8/w

Differentiating the resulting equation with respect to w and l will give;

dS/dw = l + (-8w⁻²)

dS/dw = l - 8/w²

Similarly,

dS/dl = w  + (-8l⁻²)

dS/dw = w - 8/l²

At turning point, ds/dw = 0 and ds/dl = 0

l - 8/w² = 0 and w - 8/l² = 0

l = 8/w²  and w =8/l²

l = 8/(8/l² )²

l = 8/(64/I⁴)

l = 8*l⁴/64

l = l⁴/8

8l = l⁴

l³ = 8

l = ∛8

l = 2

Hence the length of the box is 2 feet

Substituting l = 2 into the function l = 8/w² to get the eidth w

2 = 8/w²

1 = 4/w²

w² = 4

w = 2 ft

width of the cardboard is 2 ft

Since Volume = lwh

4 = 2(2)h

4 = 4h

h = 1 ft

Height of the cardboard is 1 ft

The dimensions of the box that requires the least amount of cardboard is 2ft by 2ft by 1 ft

Can someone explain to me what a “derivative” means? How do you find the derivative of f(x)=x^3+1?

Answers

The derivative is the rate of change of a function, basically represents the slope at different points. To find the derivative of the given function you can use the power rule, which means, if n is a real number, d/dx(x^n)= nx^(n-1). This is a simplification of the chain rule based on the fact that d/dx(x)=1. Anyway, this means that d/dx(x^3 + 1)= 3x^2. Here n is 3 and so it is 3*x^(3-1)= 3x^2. The derivative of x^3+1 is 3x^2.

If you are wondering what happened to the 1, for any constant C, d/dx(C)=0.

This graph shows the US unemployment rate from August 2010 to November 2011.
Sample Unemployment Rate
Graph
Unemployment Rate
10%
80%
6%
Unemployment Rate
Aug 10
Jan 11
Jun 11
Nov 11
This graph suggests unemployment in the United States
O will continue to fall.
O will continue to rise.
O will remain the same.
O will only change a little.

Answers

Answer: Will continue to rise

Step-by-step explanation:

Looking at the graph one notices that after a slight dip in the unemployment rate from August 2010 to January 2011, the unemployment rate began to rise and by November 2011 was still rising.

The arrow on the graph serves to indicate the direction the unemployment rate is going and as it is pointing upwards, this means that the Unemployment rate will continue to rise.

This was down to the fact that in 2011 the US was still yet to recover from the Great Recession of 2008 - 2009.

Answer:

EDGE 2021

Step-by-step explanation:

1) 4%

2) Increase

Factorise the following using the Difference of Two Squares or Perfect Squares rule: a) (2x-2)^2 - (x+4)^2 b) (3x+4) (3x-4)

Answers

Answer:

Step-by-step explanation:

Hello, please consider the following.

a)

[tex](2x-2)^2 - (x+4)^2 \\\\=(2x-2-(x+4))(2x-2+x+4)\\\\=(2x-2-x-4)(3x+2)\\\\=\boxed{(x-6)(3x+2)}[/tex]

b)

[tex](3x+4) (3x-4)\\\\=(3x)^2-4^2\\\\=\boxed{9x^2-16}[/tex]

Thank you.

Your friend Stacy has given you the following algebraic expression: "Subtract 20
times a number n from twice the cube of the number. What is the expression that your
friend is saying?

Answers

Answer:

Expression = 2n³ - 20n

Step-by-step explanation:

Find:

Expression

Computation:

Assume given number is 'n'

Cube of number = n³

Twice of cube = 2n³

Subtract number = 20n

Expression = 2n³ - 20n

20 POINTS! You are planning to use a ceramic tile design in your new bathroom. The tiles are equilateral triangles. You decide to arrange the tiles in a hexagonal shape as shown. If the side of each tile measures 9 centimeters, what will be the exact area of each hexagonal shape?

Answers

Answer:

210.33 cm^2

Step-by-step explanation:

We know that 6 equilateral triangles makes one hexagon.

Also, an equilateral triangle has all its sides equal.

If the tile of each side of the triangular tile measure 9 cm, then the height of the triangular tiles can be gotten using Pythagoras's Theorem.

The triangle formed by each tile can be split along its height, into two right angle triangles with base (adjacent) 4.5 cm and slant side (hypotenuse) of 9 cm. The height  (opposite) is calculated as,

From Pythagoras's theorem,

[tex]hyp^{2} = adj^{2} + opp^{2}[/tex]

substituting, we have

[tex]9^{2} = 4.5^{2} + opp^{2}[/tex]

81 = 20.25 + [tex]opp^{2}[/tex]

[tex]opp^{2}[/tex] = 81 - 20.25 = 60.75

opp = [tex]\sqrt{60.75}[/tex] = 7.79 cm  this is the height of the right angle triangle, and also the height of the equilateral triangular tiles.

The area of a triangle = [tex]\frac{1}{2} bh[/tex]

where b is the base = 9 cm

h is the height = 7.79 cm

substituting, we have

area = [tex]\frac{1}{2}[/tex] x 9 x 7.79 = 35.055 cm^2

Area of the hexagon that will be formed = 6 x area of the triangular tiles

==> 6 x 35.055 cm^2 = 210.33 cm^2


The sum of the reciprocals of two consecutive even integers is 3/4
Find the two integers.

Answers

[tex] \Large{ \underline{ \underline{ \bf{ \orange{Solution:}}}}}[/tex]

Let one of those even numbers be x, Then other even number would be x + 2.

According to question,

⇛ Their reciprocal add upto 3/4

So, we can write it as,

⇛ 1/x + 1/x + 2 = 3/4

⇛ x + 2 + x / x(x + 2) = 3/4

⇛ 2x + 2 / x² + 2x = 3/4

Cross multiplying,

⇛ 3(x² + 2x) = 4(2x + 2)

⇛ 3x² + 6x = 8x + 8

⇛ 3x² - 2x - 8 = 0

⇛ 3x² - 6x + 4x - 8 = 0

⇛ 3x(x - 2) + 4(x - 2) = 0

⇛ (3x + 4)(x - 2) = 0

Then, x = -4/3 or 2

☃️ It can't be -4/3 because it is fraction and negative number. So, x = 2

Then, x + 2 = 4

✤ So, The even numbers are 2 and 4.

━━━━━━━━━━━━━━━━━━━━


for the first one the answer are
add 5 to both sides
subtract 5 from both sides
add 1/2x to both sides
subtract 1/2 from both sides

the second one is
multiply both sides by 1/5
dived both sides by 1/5
multiply both sides by 6/7
dived both sides by 6/7

Answers

Answer:

1. add 1/2x to both sides

a. you want to combine the like terms. in this case, it is the x variable.

you are left with 7/6x = 5

2. multiply by 6/7

a. the reciprocal of 7/6 will cancel out the values

Your investment club has only two stocks in its portfolio. $25,000 is invested in a stock with a beta of 0.8, and $40,000 is invested in a stock with a beta of 1.7. What is the portfolio's beta? Do not round intermediate calculations. Round your answer to two decimal places.

Answers

Answer:

The portfolio beta is  [tex]\alpha = 1.354[/tex]

Step-by-step explanation:

From the question we are told that

      The  first investment is [tex]i_1 = \$ 25,000[/tex]

       The  first  beta is  [tex]k = 0.8[/tex]

      The second investment is  [tex]i_2 = \$ 40,000[/tex]

       The  second  beta is  [tex]w = 1.7[/tex]

Generally the portfolio beta is mathematically represented as

           [tex]\alpha = \frac{ i_1 * k + i_2 * w }{ i_1 + i_2}[/tex]

substituting values

          [tex]\alpha = \frac{ (25000 * 0.8) + ( 40000* 1.7 ) }{40000 + 25000}[/tex]

          [tex]\alpha = 1.354[/tex]

16
Select the correct answer.
If function g is defined by the equation Y-3X = -14, which equation represents the function in function notation?
OA. gx) = 3X - 14
OB. gx) = -3X - 14
OC. g(x) = 3X + 14
OD. gx) = -3X + 14
Reset
Next

Answers

Answer:  A)  g(x) = 3x - 14

Step-by-step explanation:

Solve the equation for y and replace y with g(x):

y - 3x = -14

y        = 3x - 14

  g(x) = 3x - 14

Next, the students at the Pearson Cooking Academy are assigned a take-home written exam to assess their knowledge of all things culinary. Historically, students scores on this exam had a N(68, 36) distribution. However, these days, there is an company called Charred Egg that offers to help students on tasks whether or not the exercises are for homework or for exams. In a cohort of 19 students, what is the probability that their average score will be at least 70?

Answers

Answer:

The probability is  [tex]P( \= X \ge 70 ) = 0.07311[/tex]

Step-by-step explanation:

From the question we are told that

    The  population mean is  [tex]\mu = 68[/tex]

      The standard deviation is  [tex]\sigma = \sqrt{36} = 6[/tex]

      The  sample size is  [tex]n = 19[/tex]

     

Generally the standard error of the mean is mathematically represented as  

            [tex]\sigma_{\= x } = \frac{\sigma }{\sqrt{n} }[/tex]

=>         [tex]\sigma_{\= x } = \frac{6 }{\sqrt{19} }[/tex]

=>         [tex]\sigma_{\= x } = 1.3765[/tex]

Generally the probability that their average score will be at least 70 is mathematically represented as

            [tex]P( \= X \ge 70 ) = 1 - P( \= X < 70 ) = 1 - P(\frac{ \= X - \mu }{\sigma_{\= x}} < \frac{70 - 68}{ 1.3765} )[/tex]

Generally [tex]\frac{ \= X - \mu }{\sigma_{\= x}} = z(The \ z-score \ of \ \= X )[/tex]

So

          [tex]P( \= X \ge 70 ) = 1 - P( \= X < 70 ) = 1 - P(Z <1.453 )[/tex]

From the z-table

            [tex]P(Z <1.453 ) = 0.92689[/tex]

=>          [tex]P( \= X \ge 70 ) = 1 - P( \= X < 70 ) = 1 - 0.92689[/tex]

=>         [tex]P( \= X \ge 70 ) = 1 - P( \= X < 70 ) = 0.07311[/tex]

=>          [tex]P( \= X \ge 70 ) = 0.07311[/tex]

                 

 

Suppose you are standing such that a 32-foot tree is directly between you and the sun. If you are standing 140 feet away from the tree and the tree casts a 160-foot shadow, how tall could you be and still be completely in the shadow of the tree? x 160 ft 140 ft 32 ft

Answers

Answer:

Height = 4 feet

Step-by-step explanation:

To determine how tall I can be we take the difference between the shadow cast by the 32-feet tree and the distance away from the tree

But the tree is 32 feet tall but on shadow it's 160

So lemme determine how long I'll be in my shadow first

Distance away from tree= 140 feet

Length of shadow cast by tree

= 160 feet

Length of shadow= 160-140

Length if shadow= 20 feet

My height= x

X/20= 32/160

X= 20*32/260

X = 4 feet

Height = 4 feet

PLZ answer quick i will give brainliest if right no explanation needed Joe is responsible for reserving hotel rooms for a company trip. His company changes plans and increases how many people are going on the trip, so they need at least 50 total rooms. Joe had already reserved and paid for 161616 rooms, so he needs to reserve additional rooms. He can only reserve rooms in blocks, and each block contains 8 rooms and costs $900. Let B represent the number of additional blocks that Joe reserves. 1) Which inequality describes this scenario? Choose 1 answer: a: 16+8B≤50 b: 16+8B≥50 c: 16+B≤50 d: 16+B≥50 2) What is the least amount of additional money Joe can spend to get the rooms they need?

Answers

Answer:

16 + 8b ≥ 50

4500

Step-by-step explanation:

He needs at least 50 rooms and has already reserved 16

They are in groups of 8

16 + 8b ≥ 50

Subtract 16 from each side

16+8b-16 ≥ 50 -16

8b≥ 34

Divide by 6

8b/8 ≥ 34/8

b≥ 4.25

We need to round up since we need at least 50

b = 5 since we want the least amount of rooms

Each block is 900

5*900 = 4500 more that he will have to spend

use the diagram to answer the question. AB corresponds to which line segment?

Answers

Answer:

DE

Step-by-step explanation:

I hope this helps!

An evergreen nursery usually sells a certain shrub after 9 years of growth and shaping. The growth rate during those 9 years is approximated by
dh/dt = 1.8t + 3,
where t is the time (in years) and h is the height (in centimeters). The seedlings are 10 centimeters tall when planted (t = 0).
(a) Find the height after t years.
h(t) =
(b) How tall are the shrubs when they are sold?
cm

Answers

Answer:

(a) After t years, the height is

18t² + 3t + 10

(b) The shrubs are847 cm tall when they are sold.

Step-by-step explanation:

Given growth rate

dh/dt = 1.8t + 3

dh = (18t + 3)dt

Integrating this, we have

h = 18t² + 3t + C

When t = 0, h = 10cm

Then

10 = C

So

(a) h = 18t² + 3t + 10

(b) Because they are sold after every 9 years, then at t = 9

h = 18(9)² + 3(9) + 10

= 810 + 27 + 10

= 847 cm

Other Questions
The firm has total fixed costs of $9 and a constant marginal cost of $3 per unit. The firm will maximize profit with a. 9 units of output. b. 15 units of output. c. 21 units of output. d. 30 units of output. I need helps will give you a good rating. Nicole ordered a volleyball for $9.75 Based upon what you have learned in this class, write a psychological explanation forwhat is going on in this scenario; and suggestions to help Ella improve her life.Ella is a 25 year old woman, married with two children. Her daughters are ages 1 and3. She is a university graduate and her partner brings in a good income. Over thepast six months Ella has felt tired, lost weight, and does not seem to find enjoymentin activities that she usually enjoyed.Today she awoke, sat on the couch and watched her two daughters play in the livingroom for the morning. She changed the diapers of her youngest daughter (Nora) andsent her oldest daughter (Addy) to the bathroom. After feeding them lunch, shelooked out the window. She watched as an older man fell on the side-walk, andstruggled for 10 minutes to get back on his feet. Eventually Ella lay down on thecouch and fell asleep for the afternoon.She awoke in time to make her children dinner. After feeding them and putting themto bed she had two glasses of wine while watching a movie. She felt tipsy, since shehad not eaten anything all day. Feeling exhausted, at 9 pm she went to bed for thenight. In this lesson, you've learned about many achievements and advances of ancient Mesopotamian civilizations. Choose one contribution that you think had the biggest impact on history and explain your response in about 50 to 100 words. Support your answer with evidence such as information from this lesson, previous knowledge, and outside research. A slope triangle for line l is shown on the graph below. If the slope of the line is 4/3 what is the value of w? Julissa gave out an equal number of oranges to each of the 6 apartments on her floor. if she gave each apartment 5 oranges, how many oranges did Julissa give out in all? Which of the following items would be a way to manipulate the cash flow from operating activities amount on the statement of cash flows? a. Adding depreciation back to net income to determine cash flow from operating activities. b. Including interest expense and tax expense in the calculation of cash flow from operating activities. c. Recording an item that should be recorded as an operating activity as an investing activity. d. The cash flow statement cannot be manipulated. If x is 5, then 6x = _____. please help >- Which relation is a function? In the poem Barbara Frietchie, what did Stonewall Jackson say after Barbara leaned out the window? 8 times the sum of a and b Explain your reasoning. Match each explanation to the appropriate blanks in the sentences on the right. 1. the atomic radius decreases 2. the number of gas molecules decreases 3. molar mass and structure complexity decreases 4. structure complexity decreases 5. molar mass decreases 6. each phase (gas, liquid, solid) becomes more ordered A (I_2(g), Br_2 (g), Cl_2 (g), F_2 (B): The ranking can best be explained by the trend entropy decreases as______. B (H_2O_2 (g), H_2S(g), H_2O(g): The ranking can best be explained by the decreases a trend entropy decreases as_______. C. (C(s, amorphous), C(s, graphite), C(s, diamond): The ranking can best be explained by the trend entropy decreases as_______. 4x + 5y = 19 , 5y - 4x = 38 balance sheet reports assets of $6900000 and liabilities of $2700000. All of Ivanhoes assets book values approximate their fair value, except for land, which has a fair value that is $410000 greater than its book value. On 12/31/21, Oriole Corporation paid $7030000 to acquire Ivanhoe. What amount of goodwill should Oriole record as a result of this purchase? Sally left Tampa traveling 66 mph. Keith, to catch up, left some time later driving at 75 mph. Keith caught up after 8 hours. How long was Sally driving before Keith caught up? Over the period of a few days, a hospital receives an overwhelming increase in the number of patients with food poisoning. How would a scientist respond to this increase in sick patients? A. by examining the type and source of foods each patient ate and identifying patterns of overlap within the foods consumed B. by giving the patients vitamin injections and prescribing a medication to calm their stomach and bowel symptoms C. by recording the patients' symptoms in their medical records and advising them to get rest and drink fluids D.by providing kind words, nurturing, and emotional support to the patients and asking them about their feelings The monthly cost (in dollars) of water use is a linear function of the amount of water used (in hundreds of cubic feet, HCF). The cost for using 15 HCF of water is 32.84, and the cost for using 43 HCF is 79.04. What is the cost for using 36 HCF of water? Is the exclusion of groups from rights and responsibilities oppression by force or oppression by deprivation? Which expressions are factors of the quadratic function represented by this graph?A. x and (x+6)B. (x-6) and (x+6)C. x and (x-6)D. x and -6x